Which one of the following could be an accurate and complete list of the students who review only Sunset?

AndrewArabie on September 18, 2022

Jiang can review only Sunset

In both my set up and the set up explanation there is a scenario where Jiang is reviewing only Sunset, so why isn't the the answer "Jiang, Lopez"? I understand the two can't review the same production as each other but there still exists a scenario where Jiang reviews S while Lopez reviews T/U and another where Jiang reviews T and Lopez reviews S. When I see "complete and accurate list," does that constrain me to one scenario? I've seen that terminology many times before but I don't remember it constraining me to one scenario.

Replies
Create a free account to read and take part in forum discussions.

Already have an account? log in

NHowell170 on December 31, 2022

These were my thoughts as well. Any other time that I've seen such a question, it has been asking me to choose the list of possible answers, not that the list has to be together in any one scenario. In fact, any time that I've chosen a choice outside of these parameters, I've gotten it wrong, and the explanation has been that it is a list, and not necessarily a combination of any of the possibilities.

Emil-Kunkin on January 23, 2023

Hi, I don't think we can have J in only S. I'm pretty sure that would force us to violate the final rule

Vic04324 on September 25 at 07:50PM

Untrue. In a scenario in the real-time explanation, they demonstrate a scenario where only J is in S.

It could be
S: J
T: KOM
U: ML

This, to my understanding, does not violate any rules. KO are the only two that match their reviews completely, M appears twice and LKJ each appears exactly once.

Like the others stated above, in every other answer question of this type I've encountered (which of the following is a complete and accurate list of blah blah blah), it has never been restricted to one scenario. That logic would actually lead to the wrong answer choice, as it is asking for a complete list of possibilities for something across the game. So why is JL incorrect here?

Emil-Kunkin on September 27 at 03:33AM

Ah it looks like all of us, myself included, were misreading the question stem. There are two kinds of question that use the complete and accurate list language. There are those that ask for "a complete and accurate list of..." and there are those that ask for a "complete and accurate list of ... any one of which could do blah blah blah." These actually mean two different things. The former is asking for a list of things that could do X AT THE SAME TIME, while the latter is asking for which things could do X AT ANY TIME, NOT NECESSARILY CONTEMPORANEOUSLY. This is incredibly frustrating and just one of those little things to be very very aware of. As you can see here, this regularly trips up experienced test takers and even people who think about the LSAT way too much, especially when they get sloppy as you see above.

Vic04324 on September 29 at 05:43PM

Thank you so much, Emil, this was extremely helpful!